Difference between revisions of "1967 AHSME Problems/Problem 28"

(Created page with "== Problem == Given the two hypotheses: <math>\text{I}</math> Some Mems are not Ens and <math>\text{II}</math> No Ens are Veens. If "some" means "at least one", we can conclude ...")
 
m (See also)
 
Line 13: Line 13:
  
 
== See also ==
 
== See also ==
{{AHSME box|year=1967|num-b=27|num-a=29}}   
+
{{AHSME 40p box|year=1967|num-b=27|num-a=29}}   
  
 
[[Category:Introductory Logic Problems]]
 
[[Category:Introductory Logic Problems]]
 
{{MAA Notice}}
 
{{MAA Notice}}

Latest revision as of 01:40, 16 August 2023

Problem

Given the two hypotheses: $\text{I}$ Some Mems are not Ens and $\text{II}$ No Ens are Veens. If "some" means "at least one", we can conclude that:

$\textbf{(A)}\ \text{Some Mems are not Veens}\qquad \textbf{(B)}\ \text{Some Vees are not Mems}\\ \textbf{(C)}\ \text{No Mem is a Vee}\qquad \textbf{(D)}\ \text{Some Mems are Vees}\\ \textbf{(E)}\ \text{Neither} \; \textbf{(A)} \; \text{nor} \; \textbf{(B)} \; \text{nor} \; \textbf{(C)} \; \text{nor} \; \textbf{(D)} \; \text{is deducible from the given statements}$


Solution

$\fbox{E}$

See also

1967 AHSC (ProblemsAnswer KeyResources)
Preceded by
Problem 27
Followed by
Problem 29
1 2 3 4 5 6 7 8 9 10 11 12 13 14 15 16 17 18 19 20 21 22 23 24 25 26 27 28 29 30 31 32 33 34 35 36 37 38 39 40
All AHSME Problems and Solutions

The problems on this page are copyrighted by the Mathematical Association of America's American Mathematics Competitions. AMC logo.png